Asosiy tarkibga oʻtish
Microsoft
|
Math Solver
Yechish
Amaliyot
Oʻynash
Mavzular
Algebradan oldingi
Oʻrtacha
& Usuli
Eng katta umumiy omil
Eng kam umumiy koʻphad
Operatsiyalar tartibi
Kasrlar
Aralash kasrlar
Prime Faktorizatsiya
Eksponentlar
Radikallar
Algebra
Shartlar kabi birlashtiring
O'zgaruvchi uchun yechish
Faktor
Kengaytirish
Kasrlarni baholash
Chiziqli tenglamalar
Kvadrat tenglamalar
Tengsizliklar
Tenglamalar sistemalari
Matrisalar
Trigonometriya
Soddalashtirish
Baholash
Grafiklar
Tenglamalarni yechish
Hisoblash
Derivatsiyalar
Integrallar
Chegaralar
Algebra kirishlari
Trigonometriya kirishlari
Hisoblash kirishlari
Matritsa kirishlari
Yechish
Amaliyot
Oʻynash
Mavzular
Algebradan oldingi
Oʻrtacha
& Usuli
Eng katta umumiy omil
Eng kam umumiy koʻphad
Operatsiyalar tartibi
Kasrlar
Aralash kasrlar
Prime Faktorizatsiya
Eksponentlar
Radikallar
Algebra
Shartlar kabi birlashtiring
O'zgaruvchi uchun yechish
Faktor
Kengaytirish
Kasrlarni baholash
Chiziqli tenglamalar
Kvadrat tenglamalar
Tengsizliklar
Tenglamalar sistemalari
Matrisalar
Trigonometriya
Soddalashtirish
Baholash
Grafiklar
Tenglamalarni yechish
Hisoblash
Derivatsiyalar
Integrallar
Chegaralar
Algebra kirishlari
Trigonometriya kirishlari
Hisoblash kirishlari
Matritsa kirishlari
Asosiy
algebra
trigonometriya
hisoblash
Statistika
matrisalar
Harflar
Baholash
\infty
Viktorina
Limits
\lim_{ x \rightarrow 0 } \frac{1}{x^2}
Veb-qidiruvdagi o'xshash muammolar
Showing that the \lim_{x\to 0}\frac{1}{x^2} does not exist
https://math.stackexchange.com/q/1579837
Suppose that the limit exists and equals c\in\mathbb{R}. Then for e.g. \epsilon>1 some \delta>0 must exist with \left|x\right|<\delta\implies\left|\frac{1}{x^{2}}-c\right|<1. However, if we ...
Applying L'Hopital's rule to \lim\limits_{x \to 0}\frac{2}{x^2}
https://math.stackexchange.com/questions/502024/applying-lhopitals-rule-to-lim-limits-x-to-0-frac2x2
In order to use the 0/0 case of L'Hospital's rule, we require that both the numerator and the denominator tend to 0 at the appropriate point. The numerator does not tend to 0.
Is this piece-wise function continuous, and why?
https://math.stackexchange.com/questions/2411697/is-this-piece-wise-function-continuous-and-why
If we look at the behaviour as x approaches zero from the right, the function looks like this: \begin{matrix}x & f(x) = \frac{1}{x^2} \\ 1 & 1 \\ 0.1 & 100 \\ 0.01 & 10000 \\ 0.001 & 1000000 \\ 0.0001 & 100000000\end{matrix} ...
Manipulating \lim\limits_{x \to 0}{\frac{\sqrt{x+\sqrt{x}}}{x^n}}
https://math.stackexchange.com/questions/2177214/manipulating-lim-limits-x-to-0-frac-sqrtx-sqrtxxn
If \lim\limits_{x \to 0}{\frac{\sqrt{x+\sqrt{x}}}{x^n}} = c for some c\neq 0, then \lim\limits_{x \to 0}{\frac{x+\sqrt{x}}{x^{2n}}} =c^2. Now, let \sqrt{x}=t. We then wish to find n such ...
Limit of \frac{f'(x)}{g'(x)} & g'(x) \neq 0 in Hypotheses of L'Hospital's rule.
https://math.stackexchange.com/q/110408
When we write things like \lim_{x\to a}h(x) = \lim_{x\to a}H(x) we usually mean "if either limit exists, then they both do and they are equal; if either limit does not exist, then neither limit ...
How do we calculate the Right and Left Hand Limit of 1/x?
https://math.stackexchange.com/questions/762599/how-do-we-calculate-the-right-and-left-hand-limit-of-1-x
\mathbf{Definition} : \boxed{ \lim_{x \to a^+ } f(x) = \infty } means that for all \alpha > 0, there exists \delta > 0 such that if 0<x -a < \delta, then f(x) > \alpha \mathbf{Example} ...
Ko'proq Elementlar
Baham ko'rish
Nusxa olish
Klipbordga nusxa olish
O'xshash muammolar
\lim_{ x \rightarrow 0 } 5
\lim_{ x \rightarrow 0 } 5x
\lim_{ x \rightarrow 0 } \frac{2}{x}
\lim_{ x \rightarrow 0 } \frac{1}{x^2}
Yuqoriga qaytish